Jump to content

Can you move in space?


Recommended Posts

3 hours ago, Strange said:

The force has no effect on the craft (as you admit, because nothing happens if the entire system is encased in a metal mix).

 The only effect is the momentum that leaves the craft (as you admit, because nothing happens if the entire system is encased in a metal mix).

I don’t know of any system that would work inside a metal box so I’m not sure how you are concluding that the current in the wire will have no effect on the emp or visa versa with this analogy?

Therefore your sphere analogy only references the momentum of the emp and not the magnetic field component the emp.

3 hours ago, Ghideon said:

The part above is not part of the reference you posted. It is something you added. Provide references for your claim of a ”detached force”, measured in newtons. 

If i have said ‘magnetic force’ of the emp i meant to say magnetic field, i may have used the terms interchangeably unintentionally. 

But here is what i am saying:

The emp detaches from wire A

The emp from wire A interacts with wire B

If wire B does not carry a current when the emp from wire A arrives, the magnetic field component of emp from wire A will not produce a force on wire B with formula F = I x L x B

When wire B carries a current when emp from wire A arrives, the magnetic field component of the emp from wire A which is no longer attached to wire A or the craft (detached), the magnetic field produces a force on wire B with the formula F = I x L x B 

Link to comment
Share on other sites

12 hours ago, MPMin said:

The emp leaves the wire carrying both momentum and magnetic force. You guys are analysing the momentum and not the magnetic force. It’s the magnetic force that pushes the wires and an emp is a segment of that magnetic force which becomes detached from the wire when the current is pulsed. 

No, that's not how it works.

12 hours ago, MPMin said:

F = I x L x B is the right formula to calculate the force on the wire from the magnetic force of the emp. I have disregarded the momentum of the emp as it’s effect is virtually insignificant compared to the magnetic force.

That gives you the force on a current given the presence of a magnetic field.

12 hours ago, MPMin said:

The emp’s proximity to the craft is irrelevant because it is no longer attached to the craft, further more, as I mentioned earlier in the thread, an emp can pass through non conductive substances but will interact with a wire carrying a current. In the design the wires will need to be mounted on a nonconductive material so that the magnetic forces only interact with the wires when they carry a current. 

The force is exerted on the wire. It is not something carried by the EMP, which is photons. The photons have energy and momentum.

19 minutes ago, MPMin said:

 But here is what i am saying:

The emp detaches from wire A

The emp from wire A interacts with wire B

If wire B does not carry a current when the emp from wire A arrives, the magnetic field component of emp from wire A will not produce a force on wire B with formula F = I x L x B

When wire B carries a current when emp from wire A arrives, the magnetic field component of the emp from wire A which is no longer attached to wire A or the craft (detached), the magnetic field produces a force on wire B with the formula F = I x L x B 

No current means no force from ILB, but the changing field will induce a current (or potential) from Lenz's law, which will induce a B that opposes the change in the external field.

And, as I have pointed out multiple times, when doing an analysis of this sort you can't ignore the fact that currents require closed loops. If you're only analyzing one section of the wire, you aren't going to get a valid result. You aren't in a regime where you can ignore the rest of the wire. You'll come up with silly conclusions like momentum isn't conserved.

Link to comment
Share on other sites

8 minutes ago, swansont said:

The force is exerted on the wire. It is not something carried by the EMP, which is photons. The photons have energy and momentum.

And the magnetic field produces the force on the wire when its carrying a current.  Thats how electric motors work, when you pass a current through a wire which is in a magnetic field, the wire will move, that’s why wire B will move when a current is passed through it when the magnetic field from emp A is present at wire B

 

Link to comment
Share on other sites

27 minutes ago, MPMin said:

I don’t know of any system that would work inside a metal box so I’m not sure how you are concluding that the current in the wire will have no effect on the emp or visa versa with this analogy?

1. I am NOT claiming that the current in the wire will have no effect on the emp or vice versa. You need to re-read what I wrote.

2. If, as you claim, the force on the wire is what makes the craft move, then why does this stop when in a sealed box? The wire cannot know it is in a box. The force on the wire does not change because it is in a box. So why does the force not move the craft?

Link to comment
Share on other sites

23 hours ago, Strange said:

The current in B will make no difference (which is another problem with your analysis). 

 You will get the same thrust with no current, or the current flowing in the opposite direction. The thrust is not caused by the force on the wire.

 

 

9 minutes ago, Strange said:

1. I am NOT claiming that the current in the wire will have no effect on the emp or vice versa. You need to re-read what I wrote.

What are you saying then?

 

11 minutes ago, Strange said:

2. If, as you claim, the force on the wire is what makes the craft move, then why does this stop when in a sealed box? The wire cannot know it is in a box. The force on the wire does not change because it is in a box. So why does the force not move the craft?

I say it wont work in the box because most things don’t work in a box and I don’t want a side debate as to why it would or wouldn’t work in a box with all the additional variables of the box analogy. 

Link to comment
Share on other sites

7 minutes ago, MPMin said:

I say it wont work in the box because most things don’t work in a box and I don’t want a side debate as to why it would or wouldn’t work in a box with all the additional variables of the box analogy. 

You are not answering the question... WHY wouldn't it work? The force is unchanged, so how can it not work?

This is not a "side debate"; this is key to your misunderstanding.

1. IF the force on the wire were responsible, then it would not matter if it was enclosed in a box or not.

2. It does not work in a box.

3. THEREFORE the force on the wire is not responsible for the movement.

 

Link to comment
Share on other sites

15 minutes ago, Strange said:

You are not answering the question... WHY wouldn't it work? The force is unchanged, so how can it not work?

This is not a "side debate"; this is key to your misunderstanding.

1. IF the force on the wire were responsible, then it would not matter if it was enclosed in a box or not.

2. It does not work in a box.

3. THEREFORE the force on the wire is not responsible for the movement

You are right in that I’m not answering your question about the box because you are wrong about this not being a side debate. There are too many variables to consider making this consideration inconclusive. A conventional propeller propulsion system wont work in a box ... for long but it will for a little while, will it move inside the box, maybe a little, it also depends how big the box is and does that mean it worked? Would it move the box, probably not but what if it hit the wall inside the box? Could that mean it worked? Does it mean it didn’t work even though the engine ran for a while and blew air around inside the box? In any case, just because any propulsion doesn’t work inside a box doesn’t mean it wont work outside the box so I don’t see your point?

 

Link to comment
Share on other sites

1 hour ago, MPMin said:

There are too many variables to consider making this consideration inconclusive. 

What variables?

There is only one variable, as far as I am concerned: the momentum carried away from the craft by the EMP.

There appears to be only one variable, as far as you am concerned: the force on the wire.

The first of these is changed by being in a box. 

The second is not changed by being in a box.

So, which of those two variables do you think is actually the one that matters?

1 hour ago, MPMin said:

A conventional propeller propulsion system wont work in a box ...

Irrelevant, because that is not what we are discussing.

1 hour ago, MPMin said:

so I don’t see your point?

The point is, you claim the propulsion is caused by the force on the wire. 

This force doesn't change when enclosed in a box. And yet the propulsion does. So why do you think the force is relevant to the propulsion?

 

It is all very, very simple.

Link to comment
Share on other sites

2 hours ago, MPMin said:

But here is what i am saying:

That is what you are saying, not what the references are saying. You have not yet posted a reference supporting your usage of force formula for EMP.  

Link to comment
Share on other sites

 

8 minutes ago, Strange said:

What variables?

There is only one variable, as far as I am concerned: the momentum carried away from the craft by the EMP.

There appears to be only one variable, as far as you am concerned: the force on the wire.

The first of these is changed by being in a box. 

The second is not changed by being in a box.

So, which of those two variables do you think is actually the one that matters?

Irrelevant, because that is not what we are discussing.

The point is, you claim the propulsion is caused by the force on the wire. 

This force doesn't change when enclosed in a box. And yet the propulsion does. So why do you think the force is relevant to the propulsion?

 

It is all very, very simple.

What if it does work in a box? 

Link to comment
Share on other sites

Oh my you call that an answer ?

May as well say pixies are generating your propulsion with faerie dust from their magic wands.

Try answering the question. It's extremely important for you to get what Strange is describing.

Link to comment
Share on other sites

3 minutes ago, Ghideon said:

That is what you are saying, not what the references are saying. You have not yet posted a reference supporting your usage of force formula for EMP.  

You are asking for a reference that uses the emp the way i am proposing which hasn’t been done before.

But I have posted a reference for every part of my system and each part has been supported.

3 minutes ago, Mordred said:

Oh my you call that an answer ?

 

It was a question

 

Link to comment
Share on other sites

Well I can tell you it won't work in a box. Just as Strange has been pointing out. This misconception has been pointed out numerous times in this thread in numerous different ways.

 

Edited by Mordred
Link to comment
Share on other sites

2 minutes ago, Mordred said:

Well I can tell you it won't work in a box. Just as Strange has been pointing out. This misconception has been pointed out numerous times in this thread in numerous different ways.

Can you tell me why it wont work in a box?

 

Link to comment
Share on other sites

14 minutes ago, MPMin said:

Can you tell me why it wont work in a box?

Yes.

And I have told you lots of times.

The fact that you don't understand the reason is the absolute, fundamental, key idea at the bottom of your misunderstanding.

Until you understand why it won't work in a box, you won't understand why it can work at all (which isn't the reason you think).

I think you need to figure it out for yourself before you will understand it.

Link to comment
Share on other sites

Which of the following points carries the error?

1. Wire A emits an emp 

2. The emp from wire A detaches and is no longer connected to wire A 

3. The emp from wire A carries a magnetic field 

4. The magnetic field in the emp from wire A interacts with wire B but does not interact with wire A

5. As wire B is carrying a current in the magnetic field from A, a force is produced on wire B as per F = I x L x B

6. As wire B is attached to the craft, the craft moves with wire B 

Link to comment
Share on other sites

22 minutes ago, MPMin said:

Which of the following points carries the error?

1. Wire A emits an emp 

2. The emp from wire A detaches and is no longer connected to wire A 

3. The emp from wire A carries a magnetic field 

4. The magnetic field in the emp from wire A interacts with wire B but does not interact with wire A

5. As wire B is carrying a current in the magnetic field from A, a force is produced on wire B as per F = I x L x B

6. As wire B is attached to the craft, the craft moves with wire B 

And you continue to ignore my objection about wire B needing to be a closed loop. One segment will feel a force on one direction, but for the segment with the return path, the current is in the opposite direction, the force will likewise be in the opposite direction. (simplest case, but complicating it will not save you) It will feel a torque, but not a net force.

All you'e done here is ignore a reality of the geometry that it's not valid to ignore. Current-carrying wire segments do not exist on their own.

 

Link to comment
Share on other sites

I have a problem from 2 till 6.

What is the magnetic polarity of a pulse in free air that is not supplied a continuous current ?

 Here is a couple of other questions.

What happens to Wire B after the pulse reaches the opposite side ?

How would wire B  determine its Lorentz force direction from a pulse as opposed to a continuous current. Assuming it can (see first question)( you are using the Lorentz force equations).

Ask yourself as well If you took a propellor inside a plane not outside will the plane fly ?

 

 

Edited by Mordred
Link to comment
Share on other sites

5 hours ago, swansont said:

And you continue to ignore my objection about wire B needing to be a closed loop. One segment will feel a force on one direction, but for the segment with the return path, the current is in the opposite direction, the force will likewise be in the opposite direction. (simplest case, but complicating it will not save you) It will feel a torque, but not a net force.

All you'e done here is ignore a reality of the geometry that it's not valid to ignore. Current-carrying wire segments do not exist on their own.

 

I haven’t ignored this i just haven’t addressed this yet. The wires do not necessarily have to be in a loop or coil formation. The wires could be aligned in a zig zag type formation where the wire feeds in from the top and comes out from the bottom.

 

4 hours ago, Strange said:

It is still 6

If you agree with 5 surely 6 will work too

Link to comment
Share on other sites

3 minutes ago, MPMin said:

If you agree with 5 surely 6 will work too

Nope. You started off asking about throwing rocks at a board. It still doesn't;t move the craft (unless the rock bounces off the board and leaves the back of the craft.

Do you see the connection: leaves the back of the craft.

That is what is required for a rock to make the craft move. It is also what is required for an EMP to make the craft move.

6 minutes ago, MPMin said:

The wires could be aligned in a zig zag type formation where the wire feeds in from the top and comes out from the bottom.

And those wires that come out of the top and bottom need to go somewhere. More than that, they need to form a closed loop. (Otherwise current cannot flow.)

Link to comment
Share on other sites

2 hours ago, Mordred said:

What is the magnetic polarity of a pulse in free air that is not supplied a continuous current ?

 

Considering the Polarity of a the magnetic fields when the current is continuous i dint see why it change when the current is pulse 

2 hours ago, Mordred said:

What happens to Wire B after the pulse reaches the opposite side ?

If you recall I stated that the current is also pulsed in wire B, the current would be stoped in wire B before the emp reached the other side

2 hours ago, Mordred said:

How would wire B  determine its Lorentz force direction from a pulse as opposed to a continuous current. Assuming it can (see first question)( you are using the Lorentz force equations).

The same way it does for a continuous current. The pulse is a segment of the continuous current so why would it’s orientation change?

 

2 hours ago, Mordred said:

Ask yourself as well If you took a propellor inside a plane not outside will the plane fly ?

 

If the air were able to enter through the front and exit out the back yes it could, would it be the best place for the prop, no but i also said my system would be mounted outside the craft 

Link to comment
Share on other sites

7 minutes ago, Strange said:

Nope. You started off asking about throwing rocks at a board. It still doesn't;t move the craft (unless the rock bounces off the board and leaves the back of the craft.

Do you see the connection: leaves the back of the craft.

That is what is required for a rock to make the craft move. It is also what is required for an EMP to make the craft move.

This only refers to the momentum of the emp. You haven’t addressed the magnetic field component of the emp which is what causes the force on wire B when wire B carries a current.

Link to comment
Share on other sites

Guest
This topic is now closed to further replies.
×
×
  • Create New...

Important Information

We have placed cookies on your device to help make this website better. You can adjust your cookie settings, otherwise we'll assume you're okay to continue.